Skip to Main Content

PrepTest 78, Game 3, Question 17

Transcript

This is question 17 which is the last question in PT78, Game number 3, we have another global question asking what could be true? So we'll have to go to the answer choices and just see whether they fit based on the rules. Answer choice A says S is 5, but that doesn't work because S has to come before M and V.

So you need at least two spaces for the S in this world and you need a lot more spaces for the S in this world. Answer choice B says S is fourth and that looks like it might be able to happen in this world over here. If we want to put S fourth in this world, well, the M is gonna have to come after and also the V will have to come after.

And then what we could do is we could put L first, H second, and then T third. This is an arrangement that works which proves that S could be fourth. Now, one thing you have have to be careful about if this is similar to what you've tried is that you have to put L first here. Because if you try to put H first and then L second, or H first and L anywhere else, what's gonna happen is you're gonna have to put the M before the L because of rule number 2.

And so if you didn't think B works that may be because you thought you had to put the H first, but you don't actually have to do that. You could have the L first, and that's why B is correct. Let's go ahead and eliminate the other answers, C says L is 5 and M is 6. But the issue here is if L is 5, and M is 6, you're necessarily gonna have to put the H before the L somewhere, and that's gonna trigger rule number 2.

If H is before L, then M has to be before L, it can't be after L, that's why C doesn't work. Answer choice D says T is third and L is fourth, and that might look like it works but if you try to put T in 3 in the first version of our OR, that's gonna ultimately force the S to go into 1, which contradicts the first rule. And if you're in the second version of our OR here, you could put T in 3 which means the L or the H would have to go first.

Answer choice D is saying the L is gonna go fourth, and so that means the H would have to go first cuz it can't be S. And now, you've triggered your second rule again, if H is before L, then you have to put M before L. And it looks like you could put M over there, but the M has to have an S that comes before it and now you're out of space, that's why this one doesn't work.

There's no way that you could put T in 3 and L in 4 at the same time. Answer choice E says, H is 2 and V is 3. Well, let's see, in the first world, if you tried to put H 2, well, that's not gonna work there's just way too many before H. In the second version of our OR, if you tried to put H in 2 and V in 3, what's gonna happen is S has to go before V, and that's gonna put S in 1 and S can't go in 1.

So that's why E doesn't work.

Read full transcript